LSAT and Law School Admissions Forum

Get expert LSAT preparation and law school admissions advice from PowerScore Test Preparation.

 Administrator
PowerScore Staff
  • PowerScore Staff
  • Posts: 8916
  • Joined: Feb 02, 2011
|
#27281
Complete Question Explanation

Flaw in the Reasoning—#%. The correct answer choice is (E)

This is a difficult question for many students, as only about 30% of test takers answer it correctly. The journalist’s argument is that an education party is unlikely to run. The reason? To run, a party needs at least 30% of eligible voters to either join it or donate to it, and only 26% of voters said they would join, and only 16% said they would donate. So the “join” percentage is too low, as is the “donate” percentage (both are below 30%).

Of course, what the journalist is assuming is that the 26% and the 16% overlap significantly, and therefore cannot together combine to give at least 30%. But what if the 26% and the 16% were actually different groups? For instance, if there are 100 total voters and 26 said they would join (26%), and a different 16 said they would donate (16%), then that would give a total of 42% support (26% + 16%), well above the 30% required. In that case the argument that the education party is not viable would be severely weakened.

Since the possibility clearly exists that the 26% (join) and the 16% (donate) could be separate groups, you have found a major flaw in the argument. So look for an answer choice that suggests that the joining voters may be distinct from the donating voters.

Answer choice (A): This answer choice presents an idea that strengthens the argument, as it suggests that the donating group may turn out to be even smaller than the 16% suggested.

Answer choice (B): The journalist states that it is “unlikely” that the education party is viable based on “overwhelming historical evidence,” which seems quite reasonable. The flaw is not the this party could be an exception, but rather that it is entirely possible that the party will have more support than the journalist presumes.

Answer choice (C): The argument never discusses the “amount” of money required for a party to be viable, so this cannot be thought to be a flaw/weakness. The argument is only about the total percentage of support needed.


Answer choice (D): Remember, the argument considers that over half of the voters support the education party. The problem has to do with the percentages willing to join or donate (the author believes they are too low).

Answer choice (E): This is the correct answer choice. As stated above, this answer choice suggests that some of the 16% who would donate are not included in the 26% who would join, meaning the two groups are somewhat separate (and it is more likely that a total percentage of 30% could be achieved).
 mford
  • Posts: 25
  • Joined: Aug 27, 2011
|
#1910
I think I have discovered the reason question 9 on pg. 9-6 of the full course books is so difficult. It hinges on LSAC's use of the term 'either' in the question. Like was explained earlier in the course, either in normal parlance equates to 'either, or' but either in the LSAT world can mean 'one or the other or both'. A very clever separator question. Also, the correct answer choice is worded in such a way as to seem that eligible voters that aren't prepared to join such a party would not therefore be prepared to support it (as the two concepts would otherwise appear linked on a conceptual level) therefore diminishing one both of percentages in question, and making it seem less likely that 30 % could be achieved. Good show!
 Jon Denning
PowerScore Staff
  • PowerScore Staff
  • Posts: 904
  • Joined: Apr 11, 2011
|
#5769
Agreed! Amazing how closely you have to pay attention to the subtleties/nuances in the language used. And you're right: that really is one of the most consistent tools the test makers use to separate good test takers from great ones. Glad you saw through it!
 Terry Padilla
  • Posts: 5
  • Joined: Jul 06, 2016
|
#27011
Hi there,

Could you tell me why answer choice E is the correct answer? As the question asks about something that the argument fails to consider, I assumed the correct answer would / should disprove or weaken the conclusion but I don't understand how answer choice E does that or how it is correct.

Thanks!
User avatar
 Jonathan Evans
PowerScore Staff
  • PowerScore Staff
  • Posts: 726
  • Joined: Jun 09, 2016
|
#27017
Terry,

I love this argument. Personal fave, because it illustrates effectively the issues that arise when dealing with percentages and numbers. Whenever you spot percentages of a group or numbers of a group, a flashing red light should go off in your head: "Wait a second! I bet that's going to be important, and they're going to try to throw me a curve."

Just remember this: when given different percentages, your first question needs to be, "what is this a percentage of?"

If you have percentages of different groups (high school students, college students, retired seniors, nursing home residents, arachnophobes, agoraphobes), you need to make sure that if there's a comparison involved, the base groups need to be similar in every respect.

If you have percentages from the same group, you need to ask whether the percentages overlap or not. This question illustrates this principle. You know that "only 26% would be willing to join the party" and that "only 16% would be willing to donate money to the party." The stimulus is phrased in such a way that students tend to assume (as this argument does) that the 16% who would be willing to donate to the party must also be willing to join the party. The argument assumes that at least some of those willing to donate would also be willing to join.

Here's the negation test at work:

If a sufficient number of people who were willing to donate did not in fact join the party, the party could reach the 30% threshold of support. Therefore the conclusion would be nonsensical. This is the logical flaw illustrated by answer choice E:

Some of the eligible voters who would donate money to an education party might not be prepared to join such a party.

The author has failed to consider this possibility. You correctly note that it weakens the conclusion.

Please let me know if this explanation clarifies the problem for you or if I may be of further assistance.
 Terry Padilla
  • Posts: 5
  • Joined: Jul 06, 2016
|
#27214
Hi Jon,

Thank you very much for the explaination. I believe I have the gist of it down but I want to ask a clarifying question to make sure. When I prephrased, one of the possible criticisms I thought of was the fact that some of the eligible voters who would donate could also join the party which meant that it would be LIKELY, as opposed to unlikely, that an education party was viable. Based on the negation test, is the correct answer E because it is the negated version of my prephrase and therefore logically equivalent, but just the other side of the coin?
 Adam Tyson
PowerScore Staff
  • PowerScore Staff
  • Posts: 5153
  • Joined: Apr 14, 2011
|
#27321
Terry,

I'm not sure if the admin answer posted after your question answered the question for you or not, so just in case, here goes:

I think your prephrase may have been a bit backwards. The author is apparently assuming a large overlap between the two groups - in fact he is probably assuming a total overlap, that the 16% who would donate are a subset within the 26% that would join. Looks to me like your prephrase matches that assumption, and that HELPS the argument (by showing that the party is unlikely to get 30% support). What we need to do to weaken it is show that we may indeed get 30% support. If we suggest that at least some of the donors would not join and would instead ADD TO the 26% who would join, perhaps carrying it over the 30% threshold, that hurts the argument that we will not get there.

I hope that cleared it up.
 Terry Padilla
  • Posts: 5
  • Joined: Jul 06, 2016
|
#27367
Hi Adam,

Thank you for responding to ensure that my question was answered. The admin post actually clarified it for me and helped me realize what I was missing. I attributed my assumption that the groups could be separate to the author instead of realizing that he assumed that the two groups overlapped. Because of this, I couldn't find the correct way to weaken the argument. Now, realizing that he in fact assumed that they overlapped, I understand why E is the correct answer.

Thanks for all your help with this!
 asuper
  • Posts: 10
  • Joined: Jul 21, 2018
|
#49929
Hi,
Surprisingly, I prefaced the correct answer in my head. I understood the journalist assumed the percentages overlapped and looked for an answer that shows the groups were separate which could very well reach the 30% threshold. However, when I got to E, I thought, okay this sounds right but the word "some" made me pause. "Some" implies at least one and maybe all, right? So, when I saw the answer I said to myself "well, what if some is just referring to 1% not 10%". Still wouldn't meet the thresh hold. Another instance of me overthinking. Looking back, Am i correct in saying that yes, it could mean 1%; however, the fact that it also could mean 10% is enough for it to be the correct answer? Does that make sense?
 Adam Tyson
PowerScore Staff
  • PowerScore Staff
  • Posts: 5153
  • Joined: Apr 14, 2011
|
#50105
That's correct, and you are making sense, asuper! While "some" not overlapping wouldn't prove the author wrong, his failure to consider that some might not overlap is the flaw, because that "some" could be enough to get us over the hump to 30%. If he HAD considered that some might not overlap, then he wouldn't have come to the conclusion that he did.

You got it! Super!

Get the most out of your LSAT Prep Plus subscription.

Analyze and track your performance with our Testing and Analytics Package.